Quanten.de Diskussionsforum  

Zur?ck   Quanten.de Diskussionsforum > Quantenmechanik, Relativitätstheorie und der ganze Rest.

Hinweise

Quantenmechanik, Relativitätstheorie und der ganze Rest. Wenn Sie Themen diskutieren wollen, die mehr als Schulkenntnisse voraussetzen, sind Sie hier richtig. Keine Angst, ein Physikstudium ist nicht Voraussetzung, aber man sollte sich schon eingehender mit Physik beschäftigt haben.

Antwort
 
Themen-Optionen Ansicht
  #1  
Alt 14.01.12, 16:10
baggi baggi ist offline
Newbie
 
Registriert seit: 14.01.2012
Beitr?ge: 6
Standard Winkelanteil eines Orbitals

Hallo alle miteinander,
ich mussjetz einfach mal nachfragen, weil ich bei meiner Problematik nicht vorankomme. Und zwar habe ich folgende Aufgabenstellung:
"Skizzieren Sie den Winkelanteil eines Orbitals, dessen Wellenanteil mit
Ψ(θ,φ)=(5/8*π)^0,5 *cosθ *sinφ = (5/8*π)^0,5 * x/r gegeben ist!"

In emeinen Aufzeichnungen steht, dass der Winkelanteile nochmals aufgeteilt (separiert) sind: Ψ(θ,φ) = Θ(θ) *Η (φ)

mit: Η(φ) = (2π)^0,5 * exp (imφ)
Θ(θ) = N * "legendre-Polynom" kurz LP

aber durch das LP entstehen nur Faktoren mit dem Winkel θ.
Wie komme ich also von exp (imφ) auf das sinφ ?

Danke schonmal für die Hilfe^^
Mit Zitat antworten
  #2  
Alt 14.01.12, 17:32
Hawkwind Hawkwind ist offline
Singularität
 
Registriert seit: 22.07.2010
Ort: Rabenstein, Niederösterreich
Beitr?ge: 3.076
Standard AW: Winkelanteil eines Orbitals

Zitat:
Zitat von baggi Beitrag anzeigen
Hallo alle miteinander,
ich mussjetz einfach mal nachfragen, weil ich bei meiner Problematik nicht vorankomme. Und zwar habe ich folgende Aufgabenstellung:
"Skizzieren Sie den Winkelanteil eines Orbitals, dessen Wellenanteil mit
Ψ(θ,φ)=(5/8*π)^0,5 *cosθ *sinφ = (5/8*π)^0,5 * x/r gegeben ist!"

In emeinen Aufzeichnungen steht, dass der Winkelanteile nochmals aufgeteilt (separiert) sind: Ψ(θ,φ) = Θ(θ) *Η (φ)

mit: Η(φ) = (2π)^0,5 * exp (imφ)
Θ(θ) = N * "legendre-Polynom" kurz LP

aber durch das LP entstehen nur Faktoren mit dem Winkel θ.
Wie komme ich also von exp (imφ) auf das sinφ ?

Danke schonmal für die Hilfe^^
Da assoziiere ich die Identität des Hr. Euler:

Mit Zitat antworten
  #3  
Alt 14.01.12, 17:38
Benutzerbild von Marco Polo
Marco Polo Marco Polo ist offline
Moderator
 
Registriert seit: 01.05.2007
Beitr?ge: 4.998
Standard AW: Winkelanteil eines Orbitals

sin(x)=(e^ix-e^-ix)/2i

Gruss, MP
Mit Zitat antworten
  #4  
Alt 14.01.12, 18:11
Hawkwind Hawkwind ist offline
Singularität
 
Registriert seit: 22.07.2010
Ort: Rabenstein, Niederösterreich
Beitr?ge: 3.076
Standard AW: Winkelanteil eines Orbitals

Zitat:
Zitat von Marco Polo Beitrag anzeigen
sin(x)=(e^ix-e^-ix)/2i

Gruss, MP
Da hast du ausnahmsweise mal recht.
Mit Zitat antworten
  #5  
Alt 14.01.12, 17:46
Benutzerbild von Bauhof
Bauhof Bauhof ist offline
Singularität
 
Registriert seit: 07.12.2008
Ort: Nürnberg
Beitr?ge: 2.105
Standard AW: Winkelanteil eines Orbitals

Zitat:
Zitat von baggi Beitrag anzeigen
Wie komme ich also von exp (imφ) auf das sinφ ?
Hallo baggi,

was bedeutet m?

Vielleicht hilft das weiter:

sin(x) = [exp(ix) ─ exp(─ix)] / (2i)

M.f.G. Eugen Bauhof
__________________
Ach der Einstein, der schwänzte immer die Vorlesungen –
ihm hatte ich das gar nicht zugetraut!

Hermann Minkowski
Mit Zitat antworten
  #6  
Alt 14.01.12, 17:56
Benutzerbild von Marco Polo
Marco Polo Marco Polo ist offline
Moderator
 
Registriert seit: 01.05.2007
Beitr?ge: 4.998
Standard AW: Winkelanteil eines Orbitals

Hallo Eugen,

Zitat:
Zitat von Bauhof Beitrag anzeigen
was bedeutet m?
das "im" steht für Imaginärteil.

Gruss. MP
Mit Zitat antworten
  #7  
Alt 14.01.12, 18:09
baggi baggi ist offline
Newbie
 
Registriert seit: 14.01.2012
Beitr?ge: 6
Standard AW: Winkelanteil eines Orbitals

Hallo, also danke schonmal für die Einfälle. Aber mit den angegebenen Lösungsansätzen habe ich das schon probiert. Damit komme ich nicht wirklich voran. Habt ihr noch andere Einfälle?

m ist die Magnetquantenzahl. Man betrachtet ja hier ein Orbital mit bestimmten Quantenzahlen die mit n, l, m und s gekennzeichnet sind.

Die Thetafunktion lautet wie folgt (ich weiß ehrlich gesagt nicht, wie man hier die Formeln korrekt darstellen kann):

Θ(θ) = sqrt {(2l+1)/2 * (l - |m|!)/(l+|m|!)} P "links oben steht |m|" "links unten steht l" (cos θ) also:
Θ(θ) = Normierungsfaktor* Legendre-Polynom

für m = 0 würde ja der Exponentailanteil aber auch der phi Anteil wegfallen.
mit dem Legendre-Polynom erhalte ich nur trigonometrische Funktionen von theta.
Mit Zitat antworten
  #8  
Alt 14.01.12, 19:18
Benutzerbild von Bauhof
Bauhof Bauhof ist offline
Singularität
 
Registriert seit: 07.12.2008
Ort: Nürnberg
Beitr?ge: 2.105
Standard AW: Winkelanteil eines Orbitals

Zitat:
Zitat von baggi Beitrag anzeigen
Aber mit den angegebenen Lösungsansätzen habe ich das schon probiert. Damit komme ich nicht wirklich voran. Habt ihr noch andere Einfälle?
Hallo baggi,

andere Einfälle sind vorerst nicht zu erwarten, denn wir kennen die Aufgabenstellung nicht. Ein Hinweis:

An einem Foren-Beitrag kann man eine PDF-Dateien anhängen. Bitte formuliere deine Aufgabe vollständig mit allen Parameter-Erklärungen und hänge Sie in deinem Beitrag als PDF-Datei an. Es ist mühsam, wenn man die Bedeutung der Parameter nach und nach erfragen müsste.

Du behandelst offenbar ein Spezialgebiet, das auf keinen Fall hier im Forum in die Rubrik "Schulphysik" gehört.

M.f.G. Eugen Bauhof
__________________
Ach der Einstein, der schwänzte immer die Vorlesungen –
ihm hatte ich das gar nicht zugetraut!

Hermann Minkowski
Mit Zitat antworten
  #9  
Alt 14.01.12, 21:49
Hawkwind Hawkwind ist offline
Singularität
 
Registriert seit: 22.07.2010
Ort: Rabenstein, Niederösterreich
Beitr?ge: 3.076
Standard AW: Winkelanteil eines Orbitals

Hi Eugen, ich glaube, wir sind eh durch.
Man könnte sagen, es ist eine sehr gehobene Art von Schulphysik: eine Aufgabe aus dem Kurs Quantenmechanik I, Uni-Niveau. Aber nichts wirklich spezielles: Lösung der Schrödingergleichung für das Wasserstoffatom - ein wirklich zentraler Punkt so einer Vorlesung.
Schätze, es stört niemanden, wenn die kurze Diskussion in dieser Rubrik bleibt. Könnte natürlich auch in die Quantenmechanik-Rubrik verschoben werden.

Gruß,
Uli
Mit Zitat antworten
  #10  
Alt 15.01.12, 13:54
baggi baggi ist offline
Newbie
 
Registriert seit: 14.01.2012
Beitr?ge: 6
Standard AW: Winkelanteil eines Orbitals

Hi, danke nochmals für die hilfreichen Antworten.

Die Fragestellung, die ich anfangs gepostet habe, war die Aufgabenstellung. Eigentlich war noch ein zweiter Aufgabenteil dabei: "Erklären Sie, wie man graphisch zum Wert von θ = 20° kommt" Aber das ist ja eine Kleinigkeit, weshalb ich das wegließ. Also, genauer/umfangreicher war die Frage nicht gestellt.

Da geb ich recht, dass das in diesem Forum eventuell etwas fehl am Platze ist, weil es nichts mit Schulphysik zu tun hat. Ich dachte aber auch dass es eventuell in der Quantenmechanik-, RT-Rubrik etwas falsch ist.
Ich bin Chemiestudent und es wäre nur allzuschön, wenn da nicht das Teilgebiet der theoretischen Chemie wäre ;-)
Bei der nächsten Fragestellung geht es dann in das richtige Forum (Quantenmechanik usw.)

einen schönen Sonntag,
Baggi
Mit Zitat antworten
Antwort

Lesezeichen


Forumregeln
Es ist Ihnen nicht erlaubt, neue Themen zu verfassen.
Es ist Ihnen nicht erlaubt, auf Beitr?ge zu antworten.
Es ist Ihnen nicht erlaubt, Anh?nge hochzuladen.
Es ist Ihnen nicht erlaubt, Ihre Beitr?ge zu bearbeiten.

BB-Code ist an.
Smileys sind an.
[IMG] Code ist an.
HTML-Code ist aus.

Gehe zu


Alle Zeitangaben in WEZ +1. Es ist jetzt 23:51 Uhr.


Powered by vBulletin® Version 3.8.8 (Deutsch)
Copyright ©2000 - 2024, vBulletin Solutions, Inc.
ScienceUp - Dr. Günter Sturm